what is the product of -8(9)
show your work to the problem ​

Answers

Answer 1
-72 just do 8*9 and add a negative sign
Answer 2

Answer:

-72

Step-by-step explanation:

-8×-9

=-72

hope it helps you..


Related Questions

HELPPP
A. X=20, y=10
B. X=15, y=8
C. X=20, y=12
D. X=15, y=12

Answers

Answer:

Step-by-step explanation:

If this is a parallelogram, then opposite sides are congruent and opposite angles are congruent. That means that

2y + 8 = 3y - 4 and

3x + 25 = 5x - 5

We'll solve that first equation for y:

-y = -12 so

y = 12 Now onto x:

-2x = -30 so

x = 15

Choice d is the one you want.

F(x)=sqrt x -4.find the inverse of f(x) and it’s domain

Answers

Answer:

B

Step-by-step explanation:

[tex]y = \sqrt{x } - 4 \\ \sqrt{x} = y + 4 \\ x = (y + 4) {}^{2} \\ [/tex]

Domain of inverse = Range of f = x>-4

Hence B

The correct option is B which is f⁻¹(x) = ( x + 4 )² and x ≥ -4.

What is an inverse function?

The inverse function of a function f in mathematics is a function that reverses the operation of f. If and only if f is bijective, the inverse of f is true.

A function's range is the collection of values it can take as input. After we enter an x value, the function outputs this sequence of values.

The given function is f(x) = √x - 4. The inverse function will be calculated as:-

y = √x - 4

√x = y + 4

Squaring on both sides.

x = ( y + 4 )²

Replace x with y.

y = ( x + 4 )²

The range of the function is x ≥ -4.

Therefore, the correct option is B which is f⁻¹(x) = ( x + 4 )² and x ≥ -4.

To know more about an inverse function follow

https://brainly.com/question/3831584

#SPJ5


The total profit P of manufacturing x hundred DVD's is given by the equation below.
P = 600 + 1,500x − 100x2
Find x if the total profit is $5,600.

Answers

Answer:

x = -7.5

Step-by-step explanation:

P = 600 + 1,500x − 100x^2

P = − 100x^2 + 1,500x + 600

x = -b/2a

1500/ (2 x -100)

x = -7.5


Drag each expression to the correct category.
Apply the produc rules to determine the sign of each expression.

Answers

[tex]( - \frac{4}{9} )( \frac{7}{4} ) = \: \: negative [/tex]

[tex]( - 2 \frac{3}{4} )( - 1 \frac{1}{5} ) = \: \: positive[/tex]

[tex](3)( - 3)( - 3)( - 3)( - 3) = \: \: positive[/tex]

[tex]( - \frac{1}{6} )( - 2)( - \frac{3}{5} )( - 9) = \: \: positive[/tex]

[tex]( - \frac{4}{7} )( - \frac{3}{5} )( - 9) = \: \: negative[/tex]

[tex]( - \frac{10}{7} )( \frac{8}{3} ) = \: \: negative[/tex]

Martin was selling tickets for a basketball game in a high school. He sold 1,250 tickets and the total amount collected for the game was $2,750. The student tickets cost $2 each, and adult tickets cost $3 each. How many student and adult tickets were sold?

Answers

Answer:

x = number of students tickets = 1,000

y = number of adults tickets = 250

Step-by-step explanation:

Let

x = number of students tickets

y = number of adults tickets

x + y = 1,250 (1)

2x + 3y = 2,750 (2)

Multiply (1) by 2

x + y = 1,250 (1) * 2

2x + 2y = 2,500 (3)

2x + 3y = 2,750 (2)

Subtract (3) from (2) to eliminate x

3y - 2y = 2,750 - 2,500

y = 250

Substitute y = 250 into (1)

x + y = 1,250

x + 250 = 1,250

x = 1,250 - 250

x = 1,000

x = number of students tickets = 1,000

y = number of adults tickets = 250

A post office charges 50k for a telegram of 15 words or less it charges an extra 3k for every word above 15 words find the cost of 30 words

Answers

Answer:

95k

Step-by-step explanation:

Given :

15 words or less = 50k

Every word above 15 = additional 3k

The cost of 30 words :

First 15 words = 50 k

Number of additional words = (30 - 15) = 15 words

Cost of every additional word = 3 k

Cost of 15 additional word = 15 * 3 = 45k

Total cost of 30 words :

50k + 45k = 95k

Which choice is equivalent to the product below for acceptable values of X?
Vx+2 • Vx-2

Answers

Answer:

The answer is D.

Step-by-step explanation:

Find an equation for the line below.

Answers

Answer:

y= -1/4x+ 3.75

Step-by-step explanation:

Solve |x - 5| = 7 ......

Answers

Answer:

12,-2

Step-by-step explanation:

Use the factors of the numbers to explain why
45 x 56 = 5 x 7 x 8 x 9

Answers

Answer:

45 x 56=

2520 and

5 x 7=35*8=280*9=2520

2520=2520

Hope This Helps!!!

Whoever finishes this gets "brainliest".
just for fun
35a^7--11a^7

Answers

Answer:

a=0

Step-by-step explanation:

Factor:

a^7 (35-11)

Set it equal to zero and solve:

a^7=0

a=0

Intercept Form
Point (-3,4)
Slope 5
m= b=

Answers

Answer:

y = 5x + 19

Step-by-step explanation:

y = 5x + b

4 = 5(-3) + b

4 = -15 + b

19 = b

The function f(x) is a quadratic function and the zeros of f(x) are 1 and 5. The y-intercept of f(x) is 20. Write the equation of the quadratic polynomial in standard form.

Answers

Answer:

4x² - 24x + 20 = 0

Step-by-step explanation:

a(1)² + b(1) + 20 = 0

a(5)² + b(5) + 20 = 0

a + b = - 20 ⇒ b = - a - 20 ...... (1)

25a + 5b = - 20 .......... (2)

(1) -----> (2)

25a + 5( - a - 20) = - 20

20a - 100 = - 20 ⇒ a = 4

4 + b = - 20 ⇒ b = - 24

4x² - 24x + 20 = 0

please help answer these!!!!

Jimmy’s family moved to a tropical climate. For the year that followed, he recorded the number of days that had a temperature above 400C each month. His data contained -

14, 14, 10, 12, 11, 13, 11, 11, 14, 10, 13 and 8

1) Find the mean for his data set of days that had a temperature above 400C.

2) Find the median for his data set of days that had a temperature above 400C.

3) Find the mode for his data set of days that had a temperature above 400C.

4) If, instead, there are 5 more days per month that had a temperature above 400C, what will be the mean for the data?

5) If, instead, there are 2 more days per month that had a temperature above 400C, what will be the mode for the data?

Answers

Answer:

1) 11.75

2) 11.5

3) 11 and 14

4) 16.75

5) 13, and 16

Step-by-step explanation:

The given data is 14, 14, 10, 12, 11, 13, 11, 11, 14, 10, 13, 8

1) The mean, [tex]\overline x[/tex] = (14+14+10+12+11+13+11+11+14+10+13+8)/12 = 11.75

2) The data arranged in increasing order is presented as follows;

8, 10, 10, 11, 11, 11, 12, 13, 13, 14, 14, 14

The count of he data points = 12

The median = The data at the (n + 1)/2 = (12 + 1)/2 = 6.5th position = 11 + (12  11)/2 = 11.5

The median = 11.5

3) The mode of the data set are 11 and 14

4) Where there are 5 more days per month that had a temperature above 40°C, we get;

The mean, [tex]\overline x_1[/tex] = (14+14+10+12+11+13+11+11+14+10+13+8+5×12 )/12 = 16.75 = [tex]\overline x[/tex] + 5

5) Given that there are 2 more days per month that had a temperature above 40°C, we get the days per month with temperatures above 40°C as follows;

8 + 2, 10 + 2, 10 + 2, 11 + 2, 11 + 2, 11 + 2, 12 + 2, 13 + 2, 13 + 2, 14 + 2, 14 + 2, 14 + 2

10, 12, 12, 13, 13, 13, 14, 15, 15, 16, 16, 16

Therefore, the mode are 13, and 16

please help ASAP
-
-
-
-

Answers

Answer:

The answer is A.

How I got it:

Formula of finding a triangle: 1/2 bh.

In this case, the base is R and the Height is X.

1/2rx.

You received your first credit card with a spending limit of $2,000 at 18.9% interest. You decide you need to upgrade your sound system in your car, so you are going to buy this great deal. The next month you receive your credit card statement and you now owe $2,000. You are able to pay the minimum payment of $100 each month. If you continue paying only $100 each month, how long will it take you to pay off your credit card debt?

Answers

Answer:

It will take approximately 25 months

Step-by-step explanation:

The amount owed on the credit card statement, P = $2,000

The interest rate of the credit on the credit card, r = 18.9%

The minimum monthly payment made, M = $100

The equal monthly installment formula is given as follows;

[tex]M = \dfrac{P \cdot \left(\dfrac{r}{12} \right) \cdot \left(1+\dfrac{r}{12} \right)^n }{\left(1+\dfrac{r}{12} \right)^n - 1}[/tex]

Therefore, we get;

[tex]100 = \dfrac{2,000 \cdot \left(\dfrac{0.189}{12} \right) \cdot \left(1+\dfrac{0.189}{12} \right)^n }{\left(1+\dfrac{0.189}{12} \right)^n - 1} = \dfrac{2,000 \times\left(0.01575 \right) \cdot \left(1.01575 \right)^n }{\left(1.01575\right)^n - 1}[/tex]

100×1.01575ⁿ - 100 = 31.50×1.01575ⁿ

100×1.01575ⁿ - 31.50×1.01575ⁿ = 100

68.5×1.01575ⁿ = 100

1.01575ⁿ = 100/68.5

n = ln(100/68.5)/ln(1.01575) ≈ 24.21 (which is approximately 25 months, by rounding up to the nearest whole number)

Therefore, it will take approximately 25 months to pay off the credit card debt

Find the value of x.
16.2
0.03
38.5
34.8

Answers

Hi there!

[tex]\large\boxed{x = 38.5}}[/tex]

To solve, we can use right triangle trig.

We are given the value of ∠A, and side "x" is its adjacent side. We are also given its opposite side, so:

tan (A) = O / A

tan (33) = 25 / x

Solve:

x · tan(33) = 25

x = 38.49 ≈ 38.5

Sam made $147 for 7 hours of work at the same rate how much would he make for 9 hours of work

Answers

divide $147 by 7 you’ll get 21, so he’s making 21 dollars an hour, you’ll multiply 21 by 9 which you’ll get $189
so your answer will be $189 for 9 hours of work
hope this helps!

Which expression can be used to find the difference of the polynomials?

(10m – 6) – (7m – 4)

Answers

Answer:

3m -2

Step-by-step explanation:

(10m – 6) – (7m – 4)

Distribute the minus sign

(10m – 6) – 7m + 4

Combine like terms

3m -2

convert 0.25151 to a fraction

Answers

Answer:

25151/100000

Step-by-step explanation:

Randy walks his dog each morning. he walks 7/12 of a mile in 7 minutes how many miles does he walks in 1 minutes

Answers

Answer:

1/12 mile

Step-by-step explanation:

We can use a ratio to solve

7/12 miles       x miles

---------------- = ---------------

7 minutes       1 minute

Using cross products

7 /12 * 1 = 7x

Divide each side by 7

7/12 * 1/7 = x

1/2 = x

[tex]\sf{\bold{\blue{\underline{\underline{Given}}}}}[/tex]

⠀Randy walks his dog each morning. he walks 7/12 of a mile in 7 minutes ⠀⠀⠀

[tex]\sf{\bold{\red{\underline{\underline{To\:Find}}}}}[/tex]

⠀how many miles does he walks in 1 minutes⠀⠀⠀

[tex]\sf{\bold{\purple{\underline{\underline{Solution}}}}}[/tex]

Randy walks 7/12 miles in 7 minutes

Sooo

He walks in one minutes is

7/12 miles in 7 minutes

one minutes is [tex]\sf{\dfrac{\dfrac{7}{12}}{7} }[/tex]

one minute =[tex]\sf{\dfrac{7}{12}×\dfrac{1}{7} }[/tex]

one minute=[tex]\sf{\dfrac{1}{12} }[/tex]

[tex]\sf{\bold{\green{\underline{\underline{Answer}}}}}[/tex]

Hence,

he walks in 1 minutes is 1/12 miles.

plz help me with this

Answers

0

Step-by-step explanation:

maybe

Answer:

1) 0 miles

2) (0,0)

3) the cars speed is missing? 3 * speed would be this answer

4) (3, "3*speed" )  something like (3,30) if car was going 10mph

Step-by-step explanation:

Someone tell me where everyone is going right please !!

Answers

Answer:

Everyone's action is correct except Luis

luis's equation is wrong because he flipped the inequality sign when he subtracted

true inequality ;

2.4b + 6.5 > -8.1

so Luis wrong

HAVE A NİCE DAY

Step-by-step explanation:

GOOD LUCK :D

greeetings from Turkey ツ

Answer:

(B) Luis's, because he flipped the inequality sign when he subtracted

Step-by-step explanation:

Let's evaluate each answer choice:

"Amelia's, because the variable term must be isolated on the left side"

This answer choice is wrong! Whichever side the variable term is isolated on DOESN'T affect the answer of the problem.

"Luis's, because he flipped the inequality sign when he subtracted"

This answer choice is right! Only flip the inequality sign when dividing a NEGATIVE number.

"Shauna's, because she did not apply the subtraction property of equality properly"

This answer choice is wrong! Shauna correctly applied the subtraction proper by subtracting 6.5 from both sides.

"Clarence's, because the terms he added together were not like terms"

This answer choice is wrong! Clarence added 8.1 to both sides, so he added 8.1 to 6.5, which are like terms.

Hope it helps (●'◡'●)

Given statements:
If a shape is a rhombus, then the diagonals are perpendicular.
A square is a rhombus.
What is a logical conclusion from the given statements?
OA. The sides of a square are perpendicular.
OB. The diagonals of a square are perpendicular.
OC. A rhombus is a square.
OD. The diagonals of a square are not perpendicular.

Answers

Answer:

B

Step-by-step explanation:

estimate the solution to the following system of equations by graphing.
3x+5y=14
6x-4y=9

A. ( 4/3,5/2)
B. ( 7/3, -7/2 )
C. ( -5/2, -7/2)
D. (5/2, 4/3) ​

Answers

Answer:

(101/42, 19/14)

Step-by-step explanation:

Given the following equations

3x+5y=14 ..... 1 * 2

6x-4y=9 ... 2 * 1

__________________

6x+10y=28

6x-4y=9  

Subtract

10y+4y = 28 - 9

14y = 19

y = 19/14

Get the value of x

Since 3x+5y = 14

3x+5(19/14) = 14

3x + 95/14 = 14

3x = 14 - 95/14

3x = 101/14

42x = 101

x = 101/42

Hence the solution is (101/42, 19/14)

true or false? A circle could be circumscribed about the quadrilateral below.

Answers

Answer: the answer is true

Answer:

false

Step-by-step explanation:

here the opposite angle of quadrilateral aren't supplematary the circle have no chance to be circumscribed

Water is filling a swimming pool at a constant rate. After 4 hours, 2 inches of water have filled the pool. Write an equation that gives the amount of water, w, after t hours.

Answers

Answer:

Step-by-step explanation:

Inches per hour is the rate we are looking for here, which will then be the slope of the linear equation. Slope is the same thing as the rate of change. While this may not seem all that important right now, it's actually a HUGE concept in higher math, especially calculus!

If the pool is filling at a rate of 2 inches per every 4 hours, then by dividing, we get that the rate is 1 inch every 2 hours, which translates to a slope of 1/2. Creating an equation with this slope:

[tex]w=\frac{1}{2}t[/tex]  Let's check it. We are told that after 4 hours there are 2 inches of water in the pool. That means if we plug in 4 for t and solve for w, we should get w = 2:

[tex]w=\frac{1}{2}(4)[/tex] and

w = 2. So we're good!

Evaluate i^15 i^12
Show work

Answers

Answer:  15 = -i & 12=1

use the pattern : i, -1, -i, 1

Step-by-step explanation:

i = [tex]\sqrt{-1}[/tex]

[tex]i^{2}[/tex] = [tex]\sqrt{-1}[/tex]

[tex]i^{3}[/tex] = [tex]\sqrt{-1}[/tex]

[tex]i^{4}[/tex] = [tex]\sqrt{-1}[/tex]

the pattern just repeats from here

5 = i

6 = -1

7 = -i

8  = 1

9 = i

10 = -1

11 = -i

12  = 1

13 = i

14 = -1

15 = -i

16  = 1

Apply the distributive property to factor out 5x.

(5x · x2) + (5x · 3x) − (5x · 7) =

Answers

Answer:

5x^3+15x^2-35x

Step-by-step explanation:

Firstly, you want to combine the "x"s in the (5x*x^2) and the (5x*3x). Once you have that (you will get 5x^3 and 15x^2), you can move onto the last set of parenthesis. We can get 35x from here. Finally, the last step is to add the correct signs. Our final answer will then be 5x^3+15x^2-35x. I hope this helped and please don't hesitate to reach out with more questions!

What is the explicit formula for this sequence?
7, 2, -3, -8...

Answers

Answer:

The explicit formula of that sequence is T - 5

Step-by-step explanation:

Let T represent each term in the sequence. So now try replacing T with each term in the sequence. Like this ;

7 - 5 = 2

2 - 5 = -3

-3 - 5 = -8

hope this helps

Other Questions
Canton Company sells a motor that carries a 60-day unconditional warranty against product failure. From prior years' experience, Canton estimates that 3% of units sold each period will require repair at an average cost of $160 per unit. During the current period, Canton sold 100,000 units and repaired 2,400 of those units. (a) How much warranty expense must Canton report in its cur Jill has just moved to a new town and wants to share plates of baked goods with her neighbors. She has 20 cookies and 12 brownies to share, and wants to split them equally among the plates with no food left over. What is the greatest number of plates she can make to share? QuestionMatch the elements of music to their definitions. Drag each tile to the correct box. A Brazilian steel manufacturer started selling certain categories of steel in the United States. However, the Brazilian manufacturer is selling the steel at a price significantly lower than it sells the same product back in Brazil. This practice may be a violation of U.S. law.a) trueb) false WillowWorks LLC, a retail clothing store, has been sourcing its raw materials from a supplier named ThreadTym Corp. for the last five years. A change of management at ThreadTym resulted in an increase of the prices of the raw materials. As a result, WillowWorks purchases ThreadTym so that it can produce these products on its own. This scenario best illustrates ________. Which statement is not always true for a parallelogram?A. Consecutive angles are supplementary.B. Diagonals are congruent.C. Opposite sides are congruent.D. Opposite angles are congruent. Geometry Oddsseseyware what following oxide react with both acids and bases to form salts. 17. The density of a population would influence which limiting factor?O nicheO growth rateO weatherO space Select the correct answer. Read this excerpt from the Declaration of Independence. Then answer the question that follows. We hold these truths to be self-evident that all men are created equal, that they are endowed by their Creator with certain unalienable Rights, that among these are Life, Liberty and the pursuit of Happiness. That to secure these rights, Governments are Instituted among Men, deriving their Just powers from the consent of the governed. Based on the excerpt, why do you think the Founding Fathers supported the idea of a limited government? A. They believed that it was the government's duty to fulfill the people's wishes. B. They wanted people to follow government laws and policies without conflicts. C. They felt that a limited government would reduce public revolutions and uprisings. D. They hoped to win foreign approval by following the limited government model. Find the size of angle XZY give your answer Using the approximation of 5 miles = 8 kmHow many km is 22.5 miles? The atoms in your body are mostly empty space . And so are the atoms in any wall. Why then is your body unable to pass through walls ? (06.01)Four graphs are shown below: Which graph represents a negative linear association between x and y? Which set of rational numbers is arranged from least to greatest? 1 over 5, 1.4, negative 1 over 2, 3 Identify and explainthe benefits and problems associated with high economic growth Which statement is the best description of a chemical bond Destructive interference can cause two waves to average each other out, resulting in flat water.TrueFalse Which of the following is not considered a way to demonstrate active listening? A. Consider your response while the speaker is talkingB. Encourage with verbal commentsC. Make eye contact with the speaker D. Watch the speaker's body language Factorize 5b2-11b+6=0